Ask Question
21 November, 13:09

Based on a predicted level of production and sales of 19,000 units, a company anticipates total variable costs of $70,300, fixed costs of $32,300, and operating income of $140,600. Based on this information, the budgeted amount of contribution margin for 17,000 units would be:

+3
Answers (1)
  1. 21 November, 14:14
    0
    Total Contribution margin = $154,700

    Explanation:

    Giving the following information:

    Based on a predicted level of production and sales of 19,000 units, a company anticipates total variable costs of $70,300, fixed costs of $32,300, and operating income of $140,600.

    First, we need to calculate the selling price and unitary variable cost:

    Unitary variable cost = 70,300/19,000 = $3.7

    Sales = Operating income + fixed costs + variable cost

    Sales = 140,600 + 32,300 + 70,300 = 243,200

    Unitary selling price = 243,200/19,000 = $12.8

    Now, we can calculate the total contribution margin for 17,000 units.

    Total Contribution margin = 17,000 * (12.8 - 3.7) = $154,700
Know the Answer?
Not Sure About the Answer?
Get an answer to your question ✅ “Based on a predicted level of production and sales of 19,000 units, a company anticipates total variable costs of $70,300, fixed costs of ...” in 📙 Business if there is no answer or all answers are wrong, use a search bar and try to find the answer among similar questions.
Search for Other Answers